LSAT and Law School Admissions Forum

Get expert LSAT preparation and law school admissions advice from PowerScore Test Preparation.

User avatar
 Dave Killoran
PowerScore Staff
  • PowerScore Staff
  • Posts: 5853
  • Joined: Mar 25, 2011
|
#80600
Complete Question Explanation
(The complete setup for this game can be found here: lsat/viewtopic.php?t=8622)

The correct answer choice is (A)

This question requires you to identify a list of three cities, none of which contain a hospital. Thus, for the three cities you choose, no hospitals can be assigned, and all of the hospitals in the game must be in the other three unlisted cities.

The question stem specifies that one of the cities contains an HHU block. This block must be placed in city 2 or city 3, because these are the only two cities with a university. Answer choice (C) can thus be eliminated because one of city 2 or 3 always contains two hospitals, and this answer lists both city 2 and 3 as part of a group of cities with no hospital.

Answer choice (B) can also be eliminated because we know from our initial discussion that a JH block must go in city 1, 4, or 5.

Answer choices (D) and (E) can also be eliminated because city 6 must contain a hospital, and both answer choice (D) and (E) list city 6 among the cities with no hospital.

Answer choice (A) is thus proven correct by process of elimination. In (A), the JH block would be placed in city 4, and the other two hospitals placed among some portion of cities 2, 4, or 6.
 josuecarolina
  • Posts: 24
  • Joined: Jul 20, 2012
|
#22263
Initially I had a very difficult time understanding the first rule, but got it cleared up from another post. Even after the clarification, this rule is poo. It makes it seem like there can't be a hospital, university or jail in more than one of the cities....But I get it.

Nonetheless, I did not get problem 24, and still do not understand it.

I don't even know where to start with my question, I got the others right, and they made sense, but these last 2 are just slipping from my grasp.

Thanks in advance!
User avatar
 Dave Killoran
PowerScore Staff
  • PowerScore Staff
  • Posts: 5853
  • Joined: Mar 25, 2011
|
#22264
Hey Josue,

You've run into tough questions here at the end of this section.

Question #24: Local, Could Be True, List. The correct answer choice is (A)

This question requires you to identify a list of three cities, none of which contain a hospital. Thus, for the three cities you choose, no hospitals can be assigned, and all of the hospitals in the game must be in the other three unlisted cities.

The question stem specifies that one of the cities contains an HHU block. This block must be placed in city 2 or city 3, because these are the only two cities with a university. Answer choice (C) can thus be eliminated because one of city 2 or 3 always contains two hospitals, and this answer lists both city 2 and 3 as part of a group of cities with no hospital.

Answer choice (B) can also be eliminated because we know from our initial discussion that a JH block must go in city 1, 4, or 5.

Answer choices (D) and (E) can also be eliminated because city 6 must contain a hospital, and both answer choice (D) and (E) list city 6 among the cities with no hospital.

Answer choice (A) is thus proven correct by process of elimination. In (A), the JH block would be placed in city 4, and the other two hospitals placed among some portion of cities 2, 4, or 6.

Please let me know if that helps. Thanks!
 tmmtm
  • Posts: 1
  • Joined: Jan 09, 2013
|
#7200
I cant' understand why (A) is the answer to question 24. I've considered two scenarios for "two hospitals and exactly one university" where UHH is in either CITY 2 or CITY 3.

Since J cannot appear with U or another J and both CITY 2 and 3 contain U, J is limited to being placed in either CITY 1, CITY4 or CITY 5.

In the event UHH is placed in CITY 3, CITY 2 contains U, CITY 6 contains JH and the remain J must be accompanied with H in either CITY 1, 4, or 5.

In the event UHH is place in CITY 2 the remaining JH can be placed in either CITY 1 , CITY 4, or CITY 5.

What am I missing?
 Nikki Siclunov
PowerScore Staff
  • PowerScore Staff
  • Posts: 1362
  • Joined: Aug 02, 2011
|
#7205
tmmtm,

Your reasoning is correct: the remaining J :arrow: H block needs to be placed in cities 1, 4, or 5. Keep in mind, however, that the JH is just ONE block that must occupy ONE of the three cities (1/4/5), not all three of them. This is why answer choice (A) is correct: it is possible for cities 1, 3, and 5 to contain no hospital. In the event UHH ends up in city 2, the remaining J :arrow: H can be placed in city 4. That way, the four hospitals will be distributed as follows:

2H in city 2
1H in city 4
1H in city 6

This proves that it is possible for cities 1, 3, and 5 to contain no hospital among them.

Does this help? Let me know.

Get the most out of your LSAT Prep Plus subscription.

Analyze and track your performance with our Testing and Analytics Package.